I NEED HELP ASAP. find the value of the variable that results in congruent triangles.
A. 80
B. 10
C. 50
D. 30​

I NEED HELP ASAP. Find The Value Of The Variable That Results In Congruent Triangles.A. 80B. 10C. 50D.

Answers

Answer 1

Answer:

A) 80°

Step-by-step explanation:

55° + 45° + (5x + 30)° = 180°

55° + 45° + 30° + 5x = 180°

130 + 5x = 180°

5x = 50°

x = 10°

=> (5x + 30)° = 5*10° + 30° = 50° + 30° = 80


Related Questions

NEED HELPPPP RNNNNNNNN​

Answers

put it together first so
x+12-5x=3 solve from there it easy

what do you understand by socialization​

Answers

Socialization is the lifelong process through which people learn the values and norms of a given society.

Graph the solution of the inequality 3/7(35x-14)<_ 21x/2+3

Answers

Answer:

You'll have a closed circle at x = 2, and shading to the left

See the diagram below

=========================================================

Explanation:

The fractions here are 3/7 and 21/2. The denominators of which are 7 and 2 respectively. The LCD is 7*2 = 14.

If we multiply both sides by 14, then this will clear out the denominators and make the fractions go away.

14*(3/7) = (14*3)/7 = 42/7 = 814*(21/2) = (14*21)/2 = 294/2 = 147

So if we multiplied both sides by 14, then we have these steps

[tex]\frac{3}{7}(35x-14) \le \frac{21x}{2}+3\\\\14*\frac{3}{7}(35x-14) \le 14*\left(\frac{21x}{2}+3\right)\\\\14*\frac{3}{7}(35x-14) \le 14*\left(\frac{21x}{2}\right)+14*\left(3\right)\\\\6(35x-14) \le 147x+42\\\\[/tex]

--------------------------

Let's isolate x

[tex]6(35x-14) \le 147x+42\\\\6(35x)+6(-14) \le 147x+42\\\\210x-84 \le 147x+42\\\\210x-147x \le 42+84\\\\63x \le 126\\\\x \le 126/63\\\\x \le 2\\\\[/tex]

The graph of this will consist of a closed or filled in circle at x = 2. We shade to the left to represent numbers smaller than 2.

So either x = 2 or x < 2.

If we used an open hole at 2, then we wouldn't be including 2 (but we want to include this endpoint).

See the diagram below.

Hello, Brainly community!

This question is for all of those Calculus people out there.

The volume of a swimming pool is changing with respect to time, such that the volume is given by W(t), where W(t) is measured in cubic centimeters and t is measured in seconds. A tangent line is shown for W(t) at t = 3 seconds. Determine the best estimate for the value of the instantaneous rate of change of W(t) when t = 3.
(I've narrowed down the answer choices to 2, and just really need to find the right way of thinking to find the answer)

(A) W(lim t) as t goes to 3.
(B) [W(3.1) - W(2.9)] / 0.2.

Thank you in advance!

Answers

Answer:

(B)  [tex]\displaystyle \frac{W(3.1) - W(2.9)}{0.2}[/tex]

General Formulas and Concepts:

Calculus

Limits

Derivatives

The definition of a derivative is the slope of the tangent line.

Derivative Notation

Instantaneous Rates

Tangent Line: [tex]\displaystyle f'(x) = \frac{f(b) - f(a)}{b - a}[/tex]

Step-by-step explanation:

Since we are trying to find a rate at which W(t) changes, we must find the derivative at t = 3.

We are given 2 close answer choices that would have the same numerical answer but different meanings:

(A)  [tex]\displaystyle \lim_{t \to 3} W(t)[/tex](B)  [tex]\displaystyle \frac{W(3.1) - W(2.9)}{0.2}[/tex]

If we look at answer choice (A), we see that our units would simply just be volume. It would not have the units of a rate of change. Yes, it may be the closest numerically correct answer, but it does not tell us the rate at which the volume would be changing and it is not a derivative.

If we look at answer choice (B), we see that our units would be cm³/s, and that is most certainly a rate of change. Answer choice (B) is also a derivative at t = 3, and a derivative tells us what rate something is changing.

∴ Answer choice (B) will give us the best estimate for the value of the instantaneous rate of change of W(t) when t = 3.

Topic: AP Calculus AB/BC (Calculus I/I + II)

Unit: Differentiation

Book: College Calculus 10e

Shirley buys fiction books for $20 each, and then marks up by 25% to
resell. What is the markup in dollars?

Answers

Answer:

$5

Step-by-step explanation:

Find the markup by finding 25% of 20:

20(0.25)

= 5

So, the markup is $5

find the gradients of line a and b

Answers

Answer:

Gradient of A: 2

Gradient of B: -1

Step-by-step explanation:

Gradient = change in y/change in x

✔️Gradient of A using two points on line A, (2, 5) and (0, 1):

Gradient = (1 - 5)/(0 - 2) = -4/-2

Simplify

Gradient of A = 2

✔️Gradient of B using two points on line B, (0, 5) and (5, 0):

Gradient = (0 - 5)/(5 - 0) = -5/5

Simplify

Gradient of B = -1

How many black berry trees were sampled to create this graph

Answers

Answer:

C. 31 trees

Step-by-step explanation:

Add up all the frequencies: 3+3+8+10+5+2=31

Hope this helps!!!

Answer:

It's C. 31 trees

Step-by-step explanation:

That's about all I can say

Help me to find the product plz (opt math)

Answers

Answer:

hope it helps.stay safe healthy and happy...

Answer:

[tex]\left(sin\theta -cos\:a\right)\left(cos\:a+sin\theta \right)[/tex]

(sin(θ)-cos(a))(cos(a)+sin(0))

[tex]\mathrm{Apply\:Difference\:of\:Two\:Squares\:Formula:\:}\left(a-b\right)\left(a+b\right)=a^2-b^2[/tex]

a=sin(θ),b=cos(a)

= sin²(θ)-cos²(a)

-------------------------------

hope it helps...

have a great day!!

Identify the parts of the following algebraic expression.

-8z + 1
2
y - 7.7

Term:
Variable:
Coefficient:
Constant:

Answers

Answer:

-8z+1

term:2

variable:z

coefficient:-8

constant:1

2

term:1

variable:nil

coefficient:nil

constant:2

y-7.7

term:2

variable:y

coefficient:nil

constant:-7.7

The solution is given below.

What is number?

A number is a mathematical object used to count, measure, and label. The original examples are the natural numbers 1, 2, 3, 4, and so forth. Numbers can be represented in language with number words.

now, we get,

-8z+1

term:2

variable: z

coefficient:-8

constant:1

again,

2

term:1

variable : nil

coefficient : nil

constant:2

now,

y-7.7

term:2

variable : y

coefficient : nil

constant:-7.7

To learn more on number click:

brainly.com/question/17429689

#SPJ2

Question: Dentify the parts of the following algebraic expression.

-8z + 1

2

y - 7.7

Term:

Variable:

Coefficient:

Constant:

Find the equation of a line that is perpendicular to y = -3x – 1 and passes through the point
(3,2).
Give your answer in the form y = mx + b.

Answers

the answer is:
y = 1/3x + 1
I think the answer is y= ⅓ + 1

Help!!!
Find the domain of the function. Write the answer in interval notation.

Answers

Answer:

A

Step-by-step explanation:

Given

f(x) = [tex]\frac{2}{3x+2}[/tex]

The denominator cannot be zero as this would make f(x) undefined.

Equating the denominator to zero and solving gives the value that x cannot be.

3x + 2 = 0 ⇒ 3x = - 2 ⇒ x = - [tex]\frac{2}{3}[/tex] ← excluded value

Then

domain is ( - ∞ , - [tex]\frac{2}{3}[/tex] ) U ( - [tex]\frac{2}{3}[/tex], ∞ ) → A

What is the area of a cross section that is parallel to face BFGC?

Answers

Answer: 216cm2

Step-by-step explanation: 36x6

**Does anybody knows how to do it***

instructions: state what additional information is required in order to know that the triangle are congruent for the given reason.

Given: ASA

Answers

With ASA, we need two angles and an included side. An included side is one that is between the two angles.

But, we need to make sure the sides match up. So, triangle KJI is congruent to triangle ZYX.

Therefore, we need to know that side JI is congruent to YX.

Hope this helps!

a bag contains 3 white balls and 4 red balls. A.second bag contains 4 white balls and 3 red balls. If a bag is picked at random from each bag. Find the probability that both are white

Answers

Answer:

12/49 chance that they are both white

Step-by-step explanation:

We have 2 events going on:

The first bag that contains: 3 white, 4 red

The second bag that contains: 4 white, 3 red

First let's find the probability of picking a white ball from each individual bag (not combing the chances yet):

For the 1st bag, there are 3 white balls out of 7 balls (add 3 and 4).

That means there's a 3/7 chance you'll get a white ball from the 1st bag.

For the 2nd bag, there are 4 white balls out of 7 balls (add 4 and 3).

That means there's a 4/7 chance you'll get a white ball from the 2nd bag.

To find the probability of getting both white from both of the bags, multiply 3/7 & 4/7:

3/7 * 4/7 = 12/49

You can't simplify 12/49 further, so that's your answer!

Hope it helps (●'◡'●)

Answer:

2/6

Step-by-step explanation:

_______________________________hope it helps________

the answer is... 2/6

Ava’s gross pay is $2,500 per month. Her deductions include the following:

Federal income tax $290

State income tax $85

Social Security $112

Medicare $35

What is Ava’s net pay each month?

Answers

$1,978

Explanation:

Gross pay is what’s made before taxes and necessary deductions, net pay is after taxes and necessary deductions. So we minus the deductions to find the net pay. The equation is 2,500 - 290 - 85 - 112 - 35 = ? . The answer is $1,978.

I hope this helps. Please mark me the Brainliest, it’s not necessary but I put time and effort into every answer and I would appreciate it greatly. Have a great day, stay safe and stay healthy ! :)

These points are linear.
Find the slope.
x-3 -2 -1 0 1/2
y-3 -2 -1 0 1 2
slope = [?]

Answers

Answer:

Slope = 4

Step-by-step explanation:

The slope of a linear equation can be calculated as;

m = y2-y1/x2-x1

What we have to do here is to select any two points ;

(x1,y1) = (3,8)

(x2,y2) = (6,20)

m = (20-8)/(6-3) = 12/3 = 4

URGENT PLS ANSWER QUICKLY​

Answers

Answer:

9th is 44 mark me as brainlist

Answer:

Question 9 is perimeter

Drag


the yellow point until an accurate "height" of the triangle is drawn. Afterwards, fill out the empty boxes below to determine the area of the triangle.

Answers

Answer:

I don't see a yellow point

Which of the following recursive formulas represents the same arithmetic sequence as the explicit formula an = 5+ (n-1)2?

Answers

Answer:

[tex]a_1 = 5[/tex]

[tex]a_n = a_{n-1} + 2[/tex]

Step-by-step explanation:

Given

[tex]a_n = 5 + (n - 1)2[/tex]

Required

The equivalent recursive function

The general explicit function is:

[tex]a_n = a_1+ (n - 1)d[/tex]

So, by comparison

[tex]a_1 = 5[/tex]

[tex]d = 2[/tex]

The recursion of an arithmetic sequence is:

[tex]a_n = a_{n-1} + d[/tex]

Substitute 2 for d

[tex]a_n = a_{n-1} + 2[/tex]

Hence: (a) is correct

PLZ HELP its due soon!!!

Answers

Answer:

E)

Step-by-step explanation:

1) 6.50 (starting with his current wage)

2) 6.50 + 0.25 = 6.75 $

3) 6.75 + 0.25 = 7.00 $

4) 7.00 + 0.25 = 7.25 $

5) 7.25 + 0.25 = 7.50 $

6) 7.50 + 0.25 = 7.75 $

Solve for the questions (both of them) and label you answers for which question

Answers

Jjbbh I don’t under stand

Which number completes the system of linear
inequalities represented by the graph?
y> 2x – 2 and x + 4y > _____

Answers

The system of linear inequalties that is represented by the graph is formed by the expressions y > 2 · x - 2 and x + 4 · y > - 12.

What system of inequalities generate the graph seen in the image?

Herein we have two inequalities generated by two linear functions whose form is described below:

f(x, y) > a (blue region)      (1)

g(x, y) > b (red region)      (2)

By direct comparison, we find that the limit of the red region is represented by the inequality y > 2 · x - 2. From the equation of the line we can derive an expression for the limit of the blue region:

Slope

m = [- 4 - (- 3)]/(4 - 0)

m = - 1/4

Intercept

b = y - m · x

b = - 3 - (- 1/4) · 0

b = - 3

Then, the inequation is represented by the linear equation:

y > - (1/4) · x - 3

4 · y > - x - 12

x + 4 · y > - 12

The system of linear inequalties that is represented by the graph is formed by the expressions y > 2 · x - 2 and x + 4 · y > - 12.

To learn more on inequalities: https://brainly.com/question/20383699

#SPJ1

A line with a slope of 3 passes through the point (-1, 2).

Write an equation for this line in point-slope form.

Answers

Answer:

Step-by-step explanation:

Slope = m = 3

(x₁ , y₁) = (-1 , 2)

Point slope form: y - y₁ = m(x - x₁)

y - 2 = 3(x - [-1] )

y - 2 = 3(x + 1)

y - 2 = 3*x + 3*1

y - 2 = 3x + 3

    y = 3x + 3 + 2

    y = 3x + 5

Which of the following is not a way to represent the solution of the inequality 2(x − 1) greater than or equal to −12? (1 point)

x greater than or equal to −5
x less than or equal to −5
−5 less than or equal to x
A number line with a closed circle on negative 5 and shading to the right

Answers

Answer:

x less than or equal to -5

Step-by-step explanation:

2(x-1) ≥ -12

2x - 2 ≥ -12

2x ≥ -10

x ≥ -5

PLS HELP ASAP, I need it in 10 mins. I GIVE 15 PTS !!!! if v1 = (3,-4) and v2 = (2,6) then v1*v2 is equal to which of the following?
A. 30
B. (-12, -24)
C. (6,-24)
D. -18

Answers

It is C. Good luck on ur test!!!!

v₁·v₂ is -18 which is correct option(D)

What are Arithmetic operations?

Arithmetic operations can also be specified by the subtract, divide, and multiply built-in functions.

The operator that perform arithmetic operation are called arithmetic operators .

Operators which let do basic mathematical calculations

+ Addition operation : Adds values on either side of the operator.

For example 4 + 2 = 6

- Subtraction operation : Subtracts right hand operand from left hand operand.

for example 4 -2 = 2

* Multiplication operation : Multiplies values on either side of the operator

For example 4*2 = 8

/ Division operation : Divides left hand operand by right hand operand

For example 4/2 = 2

Given that,

v1 = (3,-4) and v2 = (2,6)

To determine v₁·v₂

v₁·v₂ = 3·(2) - 4·(6)

v₁·v₂ = 6 - 24

v₁·v₂ = -18

Hence, the v₁·v₂ is -18.

Learn more about Arithmetic operations here:

brainly.com/question/25834626

#SPJ2

Quadrilateral A B C D is shown. The uppercase right angle, angle A, is 79 degrees.
What are the remaining angle measures if the figure is to be a parallelogram?

m∠B =
°

m∠C =
°

m∠D =
°

Answers

Answer:

m∠B =  

✔ 101

°

m∠C =  

✔ 79

°

m∠D =  

✔ 101

°

Step-by-step explanation:

Answer:

The answer above is right!

The correct answers are:

First box: option C. 101

Second box: option B. 79

Third box: option C. 101

Step-by-step explanation:

Just got it right on edge - Hope it helps :)

Brainliest would be greatly appreciated :D

CAN SOMEBODY PLEASE HELP MEEEE

Answers

Answer:

105.

Step-by-step explanation:

.

Can someone help with problems fivethroughseven

Answers

Answer:

5) 15120

6) 11880

7) 336

Step-by-step explanation:

The formula for permutation where mPn is m!/(m-n)!

Applying this to question 5, we get 9!/4!, which is 15120.

For question 6, we get 12!/8!, which is 11880.

For question 7, we get 8!/5!, which is 336.

Which is the area between the x-axis and y=x from x=1 to x=5

Answers

Answer:

[tex]\displaystyle A = 12[/tex]

General Formulas and Concepts:

Pre-Algebra

Order of Operations: BPEMDAS

BracketsParenthesisExponentsMultiplicationDivisionAdditionSubtractionLeft to Right

Algebra I

FunctionsFunction NotationGraphing

Calculus

Integrals

Definite IntegralsArea under the curve

Integration Rule [Reverse Power Rule]:                                                               [tex]\displaystyle \int {x^n} \, dx = \frac{x^{n + 1}}{n + 1} + C[/tex]

Integration Rule [Fundamental Theorem of Calculus 1]:                                     [tex]\displaystyle \int\limits^b_a {f(x)} \, dx = F(b) - F(a)[/tex]

Area of a Region Formula:                                                                                     [tex]\displaystyle A = \int\limits^b_a {[f(x) - g(x)]} \, dx[/tex]

Step-by-step explanation:

Step 1: Define

Identify

y = x

Interval: x = 1 to x = 5

Step 2: Sort

Graph the function. See Attachment.

Bounds of Integration: [1, 5]

Step 3: Find Area

Substitute in variables [Area of a Region Formula]:                                   [tex]\displaystyle A = \int\limits^5_1 {x} \, dx[/tex][Integral] Integrate [Integration Rule - Reverse Power Rule]:                     [tex]\displaystyle A = \frac{x^2}{2} \bigg| \limits^5_1[/tex]Evaluate [Integration Rule - Fundamental Theorem of Calculus 1]:           [tex]\displaystyle A = 12[/tex]

Topic: AP Calculus AB/BC (Calculus I/I + II)

Unit: Integration

Book: College Calculus 10e

If anyone can help with this ill mark Brainly

Answers

Answer:

In this case, we can do substitution.

Step-by-step explanation:

For the first one, (s - t)(x) = ((x - 5) - 4x^2)(x) = x^2 - 21x

For the second one, (s*t)(x) = ((x - 5) *4x^2)(x) = 4x^4 - 20x^3

And for the last one,  (s+t)(-2) = ((x - 5) + 4x^2)(-2) = -8x^2 - 2x + 10

Hope your happy with the answer :)

Other Questions
Find the area of the rhombus Marissa constructed a figure with these views.HELP ASAP EXTRA POINTS The side of a rectangle legnth x+2 and width x-5. WHich equation describes the area A of the rectangle in terms of x? Suppose that you chose sodium carbonate to precipitate the chromium ions from a solution of chromium (III) chloride. Write and balance the equation of this double-displacement reaction. If the solution has a volume of 520 mL and the concentration of chromium (III) chloride is 0.224 M, how many grams of sodium carbonate should you add to the solution to precipitate out all the chromium ions? which among the following pairs are not isotopes? What is the value of x in the equation 4 . 10 X+52 3 +35x+1 5 ?:) Which of the following is considered INCORRECT posture for a crunch?keeping your head stationarymoving until only your shoulder blades are off the groundplacing your hands behind your head or neckkeeping your legs straight out in front of you He's very short: ________ sisters are taller. (A) both of them (B) his both (C) both his (D) the two both his A conservation organization conducted several negotiations with the government to prevent the dissemination of mercury in several food products. Since the negotiations with the government were unsuccessful, the organization prepared a press release and used its website, social media, and email to reach its audiences to support its cause. The term used to define the organization's approach is ________ lobbying because it resorted to indirect tactics to achieve its goal. Joe's behavior is a result of the negative thoughts he has about his job performance and his own abilities. His internal dialogue is causing his self-worth to be damaged and is affecting his work. If his thoughts could be changed, and he stopped beating himself up, his performance would improve. Which psychological approach best fits the explanation: a Behaviorist b Socio-Cultural c Psychoanalytic d Humanist e Cognitive f Biological Find the volume of the solid obtained by rotating the region bounded by the given curves about the specified axis: y=x6, y=1 about y=6. The equations x minus 2 y = 4, 4 x + 5 y = 8, 6 x minus 5 y = 15, and x + 2 y = 0 are shown on the graph below.On a coordinate plane, there are 4 lines. Green line goes through (0, 0) and (2, -1). Blue line goes through (0, negative 2) and (2, negative 1). Pink line goes through (0.75, negative 2), and (2.5, 0). Purple line goes through (0, 1.5) and (2, 0).Which system of equations has a solution of approximately (1.8, 0.9)?6 x minus 5 y = 15 and x + 2 y = 04 x + 5 y = 8 and 6 x minus 5 y = 15x minus 2 y = 4 and 4 x + 5 y = 86 x minus 5 y = 15 and x minus 2 y = 4 Suppose a 48. L reaction vessel is filled with 1.6 mol of Br2 and 1.6 mol of OCl2. What can you say about the composition of the mixture in the vessel at equilibrium what would happen if you boil a substance in an alcohol The quantity demanded x for a product is inversely proportional to the cube of the price p for p > 1. When the price is $10 per unit, the quantity demanded is 64 units. The initial cost is $140 and the cost per unit is $4. What price will yield a maximum profit? (Round your answer to two decimal places.)$______ A shipment of 50 precision parts including 4 that are defective is sent to an assembly plant. The quality control division selects 10 at random for testing and rejects the entire shipment if 1 or more are found defective. What is the probability this shipment passes inspection? A strawberry farmer in Poteet knows that 1/8 of his strawberries are typically not fit to sell at the market (either because they went bad or are too unusually shaped). The farmer takes a random sample of 156 strawberries to inspect for the upcoming farmer's market and finds that 24 are unfit to sell. If he were to go back and pick 1000 more strawberries to inspect for the market, how would the standard deviation of the sample proportion be affected What is a synonym for the word enjoy in this sentence?Many people neither enjoy nor _____ modern art.A. EnvyB. AppreciateC. Delight inD. ViewE. There is no appropriate synonym. one car accelerates at half the rate of another how much longer does it take the first car to travel a quarter mile What do Macbeth's conflicting emotions about the murder of Duncan suggestabout his character?